CIC Practice Exam 3

Ace your homework & exams now with Quizwiz!

29. The air exchanges in the decontamination area of Sterile Processing should be negative with a minimum of how many air exchanges per hour? a. Six b. Eight c. Nine d. Ten

29. D Ten Rationale: The ventilation of the decontamination area should be negative air pressure (pulls air into the work area) with 10 air exchanges/hour and all air exhausted to the outside atmosphere.

39. During outbreaks, the CDC recommends that high-touch surfaces in patient rooms be cleaned and disinfected: a. At least three times a day b. Hourly c. At least twice a day d. Only when soiled

39. A Three times a day Rationale: During outbreaks, the environment, especially high-touch surfaces, is heavily contaminated with microorganisms. High-touch surfaces include commodes, toilets, toilet handles, faucets, bathroom rails, bedrails, telephones, computers, and food preparation areas. The CDC recommends that these surfaces be cleaned and disinfected at least three times a day and that low-touch surfaces be cleaned at least twice a day.

31. Which of the following is not a component of the Needlestick Safety and Prevention Act?a. Provide safety-engineered sharps devices and needleless systems to employees to reduce the risk of occupational exposure to bloodborne diseases b. When selecting safety-engineered products, solicit input from employees who provide direct patient care and have a high risk of injuries from contaminated sharps c. Maintain a sharps injury log to record injuries from contaminated sharps d. Require the Hepatitis A vaccine for all employees who are at risk for occupational exposure

1. D Require the Hepatitis A vaccine for all employees who are at risk for occupational exposure Rationale: Hepatitis A is spread person to person; occupational exposure generally does not increase HCP risk for Hepatitis A virus (HAV) infection. To be compliant with the Needlestick Safety and Prevention Act, healthcare facilities should:• Provide safety-engineered sharps devices and needleless systems to employees to reduce the risk of occupational exposure to bloodborne diseases.• Solicit input from employees responsible for direct patient care who are potentially exposed to injuries from contaminated sharps in the identification, evaluation, and selection of effective safety-engineered products and work practice controls.• Document the solicitation of input in the exposure control plan.• Maintain a sharps injury log to record injuries from contaminated sharps. The injury log must contain the type and brand of product involved in the incident, the work area where the incident occurred, and an explanation of how the incident occurred

100. Which of the following has a low risk of transmission from sexual contact? a. Hepatitis B b. Hepatitis C c. HIV d. Syphilis

100. B Hepatitis C Rationale: HCV is inefficiently transmitted by sexual intercourse (prevalence ranging from 1.3 percent in North America to 27 percent in Asia in long- term partners), though homosexual men, persons with multiple sexual partners and HIV patients have higher rates of seroprevalence for HCV than monogamous heterosexuals. The average risk for vertical transmission is 6 percent overall and 17percent in mothers with HIV, which appears to be related to viral titer. No difference in transmission is noted whether the child is breast- or bottle-fed.

101. Of the following sharp object injury examples, which would have the highest risk of transmission of bloodborne pathogens (assuming that all patient-related risk factors are identical for each example)? a. A nurse is stuck with an intravenous (IV) catheter stylet after withdrawing the stylet from the catheter b. A medical resident is stuck with a suture needle that had been used to suture a head wound c. A nurse is stuck with the needle from a syringe that had been used to give an intramuscular injection d. A surgeon sustains a superficial skin injury from a used disposable scalpel

101. A A nurse is stuck with an intravenous (IV) catheter stylet after withdrawing the stylet from the catheter Rationale: IV catheter stylets are involved in only about 3 percent of sharp object injuries, but they have the highest risk of transmission of bloodborne pathogens because they are hollow-bore needles that can be filled with blood.This results in greater exposure to bloodborne pathogens.

103. There is a suspected case of measles in the ED of a facility, and the patient has been admitted. A nasopharyngeal swab was taken, placed in viral transport media, and sent to the lab where it was frozen at -20°C for 12 hours and then thawed and placed in culture. The culture results are negative for measles virus. Which of the following should the IP request for this patient? 1) A new sample should be collected and placed in a -20°C environment immediately 2) A new sample should be collected and placed in culture immediately 3) The patient should be placed in an airborne infection isolation room 4) The patient should be placed in a standard room without isolation precautions a. 1 b. 4 c. 2, 3 d. 1, 2, 3

103. C 2, 3 Rationale: Measles, or Rubeola, virus is a temperature labile virus that should be transported on ice to the lab as soon as possible after collection and placed in culture immediately or frozen at -70°C until being placed in culture. Measles virus samples should not be kept at room temperature or frozen at -20°C because these temperatures will lower the infectivity of the virus in the sample and this could produce a false negative result. The patient must be placed in an airborne infection isolation room because there is a clinical suspicion of measles and the initial negative test was not performed properly.

104. Measures of central tendency are: a. Ratios and rates b. Proportions and standard deviation c. Mean and median d. Percentiles and measures of dispersion

104. C Mean and median

105. ICRA elements related to building site areas affected by construction include all of the following except: a. Impact of potential outages or emergencies and protection of patients during planned or unplanned outages, movement of debris, traffic b. the number of containment cubes owned by the facility c. impact of disrupting essential service to patients and employees d. determination of specific hazards and the protection levels needed for each

105. B The number of containment cubes owned by the facility Rationale: An ICRA must guide a strategic, proactive design to mitigate environmental sources of microbes, to prevent infectious hazards through architectural design (e.g., hand washing and hand hygiene stations, isolation rooms; materials selection for surfaces and furnishings) and to provide control measures that mitigate potential contamination during actual construction or renovation (e.g., dust barriers, pressure differentials, protection of air handlers).

06. The Infection Prevention Manager has been directed to design a new system for housewide surveillance of CLABSI in a 300-bed urban community teaching hospital. Which of the following tools would be most appropriate to help ensure that all aspects of this large-scale project are addressed? a. A detailed contingency plan b. A summary of the scope of work c. A quality management plan d. A work breakdown structure

106. D A work breakdown structure Rationale: A work breakdown structure (WBS), is the decomposition of a project into smaller components. Elements of the plan may be a product, data, service, or any combination thereof. A WBS also provides the necessary framework for detailed cost estimating and control along with providing guidance for schedule development and control.

107. Which of the following would be appropriate for a graph displaying C. difficil infections? a. X axis labeled with the months of the year b. X axis labeled with the number of cases each month c. Y axis labeled with the months of the year d. Y axis labeled with the days of the month of April

107. A X axis labeled with the months of the year Rationale: Graphs are a method of showing quantitative data using a system of coordinates. A well-constructed graph consists of two sets of lines that intersect at right angles. Each axis (line) has a scale measurement and a label. Time (year, month, quarter, day, etc.) is usually represented on the horizontal (x) axis. The vertical (y) axis usually reflects the frequency of occurrence of an event (e.g., the number of cases of disease) or the proportion (e.g., percent, cases per 1,000 patient days) with the event. Each graph should be simple and self-explanatory.

108. When transporting used instruments from the operating room to the decontamination area, they must be transported in: a. An open bin b. Large plastic bags c. Puncture-proof, sealable containers labeled as biohazardous d. A bin draped with a sheet

108. C Puncture-proof, sealable containers labeled as biohazardous Rationale: Contaminated items should be placed in puncture-proof, sealable containers and visibly labeled biohazardous. The selection of the container will depend on the size, presence of sharps, configuation, and volume/numbers of medical devices or instruments being transported.

109. The IP is notified of a positive Legionella test in an individual who has resided in the healthcare facility for 2 months. Which of the following are potential sources of Legionella that should be investigated? 1) Cooling towers 2) IV solutions 3) Air conditioners 4) Ice machines a. 1, 4 b. 1, 3 c. 3, 4 d. 2, 4

109. A 1, 4 Rationale: L. pneumophila is a common cause of both community-acquired and healthcare-associated pneumonia. Clinical manifestations are nonspecific,but high fever, diarrhea, and hypernatremia are common. Infection has been linked to drinking water distribution systems of acute care and extended care facilities. When a patient receives a diagnosis of healthcare-associated Legionnaires' disease, culturing of the water sites to which the patient was exposed is indicated. Distal sites include water faucets, ice machines, water used in respiratory tract devices, and water sources to which the patient is exposed. Air conditioners have not been implicated in Legionnaires' disease. IV solutions are not known to be sources of Legionella infections.

110. OSHA requires the use of engineering controls to prevent transmission of bloodborne pathogens in the healthcare setting. Which of the following is an example of an engineering control?a. Requiring Hepatitis B vaccination for all HCP with occupational exposure b. Providing eye protection for all personnel who have potential for exposure to bloodborne pathogens c. Implementing the use of Universal Precautions d. Purchasing self-sheathing syringes

110. D Purchasing self-sheathing syringes Rationale: Engineering controls prevent transmission of bloodborne pathogens. These controls use technology to reduce and remove the potential for contact with sharp objects that may be contaminated with blood and body fluids. Eamples of engineering controls include safety needles, retractable scalpels, and sharps containers. Engineering controls are only valuable if they are used correctly. Education about correct use of the technology is a key factor in reducing exposure to bloodborne pathogens.

111. The IP is teaching a group of ICU nurses about the risk factors for healthcare-associated infection (HAI) during patient care. Which of the following are factors that may influence the infectious risk? 1) Type of patient care activity 2) Mode of transmission of an infectious agent 3) Patient's host defenses 4) Patient's past surgical history a. 1, 2, 3 b. 2, 3, 4 c. 1, 3, 4 d. 1, 2, 4

111. A 1, 2, 3 Rationale: The risk of HAI during patient care is related to the mode of transmission of the infectious agent, the type of patient care activity or procedure being performed, and the individual's underlying host defenses. The duration of exposure, inoculum, and pathogenicity of the infectious agent also significantly influees the infection risk.

112. Which of the following laboratory methods is utilized for viral testing? a. Gram staining, to quickly confirm presence of infection b. Broth dilution, to determine antimicrobial resistance via minimal inhibitory concentration (MIC) c. Antibody assay, to detect viral antibodies in the serum d. Antibody assay, to detect viral antibodies in clinical specimen

112. C Antibody assay to detect viral antibodies in the serum Rationale: There are three major methods to diagnose viral infections: direct detection in the clinical specimen, specific antibody assay to detect viral antibodies in the serum, and viral culture. Gram staining and broth dilution are used for bacterial pathogens. Antibody assays are performed on serum.

113. Seventy-fie patients were admitted to the Medical-Surgical ICU. Forty were on the surgical service and 35 were on the medical service. Fifteen patients developed a HAI with MRSA. Nine of the patients with MRSA infection were on the surgical service. There were 230 patient days in the ICU for the surgical patients in January, and 325 patient days for medical patients. What was the MRSA attack rate for patients on the medical service? a. 8 percent b. 2 percent c. 17 percent d. 15 percent

113. C 17 percent Rationale: An attack rate is a special form of incidence rate. It is not truly a rate, but a proportion. It is the proportion of persons at risk who become infected over an entire period of exposure or a measure of the risk or probability of becoming a case. It is usually expressed as a percentage and is used almost exclusively for epidemics or outbreaks of disease where a specific population is exposed to a disease for a limited period of time. The attack rate equals the number of new cases of disease (for a specified time period)divided by the population at risk for the same time period multiplied by 100. Attack rate is the same as incidence rate, except that attack rates are always expressed as cases per 100 populations or as a percentage.

114. A dialysis patient has recently tested positive for HBsAg. Previous tests have been negative. The manager is concerned that the patient may have been exposed during a treatment. Which of the following is the most likely explanation of the patient's positive test result? a. The patient is resolving an acute Hepatitis B infection b. The patient received a dose of Hepatitis B vaccine in the last 21 days c. The result is inaccurate d. The patient is immune due to natural infection

114. B The patient received a dose of Hepatitis B vaccine in the last 21 days Rationale: HBsAg is a protein on the surface of HBV; it can be detected in high levels in serum during acute or chronic HBV infection. Care should be taken when testing for HBsAg because recent administration of Hepatitis B vaccine may result in positive HBsAg results for 7 to 30 days following vaccination.

115. A patient is admitted with fever, rash, headache, abdominal pain, vomiting, and muscle pain. The IP is aware that Rocky Mountain spotted fever is endemic in the area. The patient reports a recent camping trip with exposure to ticks. Which of the laboratory tests listed below should be ordered to detect and differentiate the appropriate antibodies in the serum? a. Blood culture b. Weil-Felix agglutination c. Sedimentation rate d. Cold agglutinin

115. B Weil-Felix agglutination Rationale: Rocky Mountain spotted fever (RMSF) is a tickborne disease caused by the bacterium Rickettsia rickettsii. This organism is a cause of potentially fatal human illness in North and South America and is transmitted to humans by the bite of infected tick species. Weil-Felix agglutination is a test performed to differentiate rickettsial antibodies in the serum. This test can be useful in diagnosing RMSF.

40. A patient has been admitted to a healthcare facility with Neisseria meningitidis and placed on Droplet Precautions. When can the patient be removed from Droplet Precautions? a. After three consecutive CSF cultures are negative for N. meningitidis b. After the signs and symptoms of meningitis have resolved c. 24 hours after initiation of effective therapy d. The patient should remain on Droplet Precautions for the duration of the hospital stay

40. C 24 hours after initiation of effective therapy Rationale: The patient must remain on Isolation Precautions for 24 hours after appropriate antimicrobial therapy has been initiated.

116. Breaches in patient safety are being studied to determine the cause and effect of human error. Human factors engineering is a tool that: a. Studies processes to achieve "failure-free" operation over time to reduce defects and improve system safety b. Studies people at work, then designs tasks and the working environment so that people can be safe, effective, and productive c. Studies human characteristics and is concerned with design of tools, machines, and systems that take into account human capabilities d. Studies the elements involved with human-machine interface to improve working conditions

116. C Studies human characteristics and is concerned with design of tools, machines, and systems that take into account human capabilities Rationale: Human factors engineering (HFE) involves research in human psychological, social, physical, and biological characteristics and is concerned with design of tools, machines, and systems that take into account human capabilities, limitations, and characteristics. The goal is to create designs that are safe, comfortable, and effective for humans to use.

117. Immunizations recommended for all HCP include: 1) Human papillomavirus vaccine (HPV) 2) Shingles (herpes zoster) vaccine 3) Influenza accine 4) Tetanus, diphtheria, and pertussis (Tdap) vaccine a. 1, 4 b. 2, 3 c. 3, 4 d. 2, 4

117. C 3, 4 Rationale: The ACIP recommends that all HCP regardless of their age receive a single dose of Tdap if they have not been previously vaccinated. ACIP also recommends that all HCP receive seasonal influenza accine annually.

118. Which classroom setup best promotes interaction between learners and teachers? a. Stadium or auditorium style with clear sight lines and good acoustics b. Rectangular conference tables that promote a more formal space for learning c. Classroom style with straight rows of desks that also provide writing surfaces d. A horseshoe shape that also provides writing surfaces

118. D A horseshoe shape that also provides writing surfaces Rationale: A horseshoe shape allows face to face participant contact while allowing the educator and equipment to be easily positioned for visibility. Whatever classroom setup is used, efforts should be made to encourage interaction via chair placement or grouping of participants.

12. A 16-year-old male is brought to the ER with a fever, and a wound on his left leg that is draining purulent material. He complains of swelling and pain. He is a quarterback on the high school football team and has several turf burns on both legs. Past medical history is insignificant. he culture grows gram-positive organisms in clusters. What is the most likely differential diagnosis? a. Parasitic infection b. Atypical mycobacterium infection c. Methicillin resistant Staphylococcus aureus d. Streptococcus pyogenes

12. C Methicillin resistant Staphylococcus aureus Rationale: Athletes can contract CA-MRSA by close skin to skin contact, sharing athletic equipment, sharing towels, razors, or other personal items with someone who has an active infection or who is a carrier. Athletes may get abrasions from artificial turf; these abasions may serve as an entry point for CA-MRSA. In the community, most MRSA infections are skin infections that may appear as pustules or boils which often are red, swollen, painful, or have pus or other drainage. They often firt look like spider bites or bumps that are red, swollen, and painful. These skin infections commonly occur at sites of visible skin trauma, such as cuts and abrasions, and areas of the body covered by hair.

120. The Joint Commission National Patient Safety Goal (NPSG) 7 focuses on the prevention of infections. The IP has been asked to help identify what education should be provided to the patients regarding prevention of SSI, CLABSI, CAUTI, and management of multidrug-resistant organisms (MDROs). What recommendation is applicable to all of these issues and would apply to all patients receiving care in the hospital? a. Cough containment to reduce transmission of airborne pathogens b. Importance of hand hygiene and asking caregivers if they have washed their hands c. Mode of transmission of microorganisms from one area of the body to another d. Symptoms associated with infection and the need to report them to healthcare providers

120. B Importance of hand hygiene and asking care givers if they have washed their hands Rationale: The Joint Commission's National Patient Safety Goal 7 emphasizes the importance of patient education concerning key topics such as SSI prevention, prevention of central line-associated infections, and the management of resistant organisms. All patients should be taught the concepts of hand hygiene, including when to use soap and water or alcohol-based hand rubs. Patients should also be taught to ask their care givers if they have washed their hands prior contact with the patient.

121. The viral load in a patient with HIV: a. Increases during the antiretroviral phase, decreases during asymptomatic HIV infection, then increases as the patient progresses to acquired immune deficieny syndrome (AIDS) b. Remains low during the antiretroviral phase, increases during asymptomatic HIV infection, then decreases as the patient progresses to AIDS c. Remains low during the antiretroviral phase, increases during asymptomatic HIV infection, then increases as the patient progresses to AIDS d. Increases during the antiretroviral phase, decreases during the asymptomatic HIV infection, and continues to decrease as the patient progresses to AIDS

121. A Increases during the antiretroviral phase, decreases during asymptomatic HIV infection, then increases as the patient progresses to acquired immune deficiency syndrome (AIDS) Rationale: During the acute retroviral syndrome there is a high level of HIV viremia with plasma HIV RNA titers of 105 copies per mL. Therefore, when a patient presents with signs and symptoms compatible with acute retroviral syndrome, the laboratory diagnosis is based on the determination of HIV RNA titers or viral load, not determination of HIV serology.

122. What type of surveillance is the monitoring of bloodstream infection rates? a. Outcome surveillance b. Mandatory reporting c. Process surveillance d. Combined surveillance

122. A Outcome surveillance Rationale: Surveillance programs should measure outcomes of healthcare, processes of healthcare, and selected events of importance to the healthcare organization. Examples of outcome indicators that may be monitored include HAIs (e.g., bloodstream, urinary tract, pneumonia, surgical site, conjunctivitis, upper respiratory tract, or local intravenous site); infection or colonization with a specific organism (e.g., C. difficil, MRSA, VRE, or other MDRO, RSV, or rotavirus); decubitus ulcers; phlebitis related to peripheral intravascular therapy; pyrogenic reaction or vascular access infection in hemodialysis patients; resident or patient falls; influenza or ST conversions in patients, residents, or HCP; and sharps injuries and blood/body flud exposures in HCP.

123. A patient in the Neurosurgical ICU develops a fever. Cultures are ordered and collected. The physician decides to start an antibiotic while waiting for the culture results because the patient is critically ill. This type of antibiotic usage is called: a. Empiric b. Prophylactic c. Therapeutic d. Pathogen-directed

123. A Empiric Rationale: When no definitive information about a causative pathogen is available (though Gram stain can be highly suggestive), therapy is said to be empirical. Typically, hospitalized patients are sufficiently ill o warrant treatment before culture and sensitivity results are available, and therapy while the results of cultures are pending may represent most empirical therapy. Especially in hospitalized patients, appropriate cultures, usually including more than one blood culture, should be collected before the initiation of therapy. The site of infection determined clinically (e.g., lung, urinary tract) and host factors (e.g., HIV, organ transplant patient) give an indication of likely pathogens and should shape the decision regarding empirical therapy. Empirical therapy, compared with pathogen-directed therapy, is broader in spectrum due to uncertainty about the causative agent.

124. The IP has completed a series of education programs and summarized both the mean and standard deviation for four groups of participants. Which set of scores indicates the most consistent level of performance among the attendees? a. Group 1: Mean 88 SD 6.4 b. Group 2: Mean 87 SD 3.5 c. Group 3: Mean 90 SD 15.8 d. Group 4: Mean 92 SD 20.3

124. B Group 2: Mean 87 SD 3.5 Rationale: Standard deviation is a measure of dispersion of the raw scores that reflects the variability in values around the mean. It employs the squared deviations from the mean (variance), which therefore gives added emphasis to larger deviations. The standard deviation indicates how small the variability is (i.e., the spread) among observations. If the variability is small, all the values are close to the mean. If it is large, the values are not close to the mean. Group 2 has the smallest standard deviation, indicating less variability and thus greater consistency among the scores.

125. The Infection Prevention Director is revising roles within the infection prevention team to better utilize individual skills and increase the effectiveness of the infection prevention and control program. By exploring ways to add responsibilities, to use additional skills and abilities, and to include more recognition, the director is accomplishing: a. Job enrichment b. Job enlargement c. Job intensifiction d. Job rotation

125. A Job enrichment Rationale: Job enrichment is defined as a way to motivate employees by giving them more responsibilities and variety in their work. The concept was developed by American psychologist Frederick Herzberg in the 1950s. According to Herzberg, a well-enriched job should contain a range of tasks and challenges of varying difficulties, meaningful tasks, and eedback, encouragement, and communication. Allowing employees more control over their work can stimulate their desire to succeed.

126. There is a shortage of influenza accine, and the IP has been asked to help prioritize the administration of the vaccine among HCP in the facility. Which of the following employee groups would be among the highest priority for immunization? a. The admissions clerk in the ED b. The Lung Transplant Coordinator c. A nurse in labor and delivery d. The CEO of the hospital

126. B The Lung Transplant Coordinator Rationale: Of all HCP listed, this employee would get priority because they work postoperatively with immunocompromised patients who are at high risk for influenza infection. Any HCP who work with high-risk patients should be vaccinated against influenza

127. An outbreak of norovirus in a LTCF would most likely have an epidemic curve (epi curve) that: a. Indicated a common source of infection b. Indicated a propagated source of infection c. Indicated a point source of infection d. Indicated a common vehicle of infection

127. B Indicated a propagated source of infection Rationale: An epidemic curve gives a graphical display of the numbers of incident cases in an outbreak or epidemic, plotted over time. The form of the resulting distribution of cases can be used to propose hypotheses on the nature of the disease and its mode of transmission.A propagated source means that infections are transmitted from person to person in such a way that cases identified cannot be ttributed to agent(s) transmitted from a single source. Propagated (continuing) source cases occur over a longer period than in common source transmission. Explosive epidemics resulting from person-to-person transmission may occur (e.g., chickenpox). If secondary and tertiary cases occur, intervals between peaks usually approximate average incubation period.

128. What healthcare-associated viral outbreaks are most frequently reported among infants and children? a. Hepatitis A b. Coxsackie A c. Staphylococcus aureus d. RSV

128. D RSV Rationale: RSV is one of the most important causes of respiratory tract infection in infants and the elderly worldwide. It is transmitted by direct and indirect contact. RSV is a major preventable HAI with frequent outbreaks that can lead to high mortality rates in healthcare facilities. Proper infection prevention measures, including hand hygiene, Standard and Contact Precautions, cohorting, and rapid diagnostic techniques are critical in controlling the spread of RSV in healthcare facilities.

129. Which of the following is not confirmtory of an active measles infection? a. Positive measles immunoglobulin M (IgM) b. Fourfold increase in measles immunoglobulin G (IgG) c. Positive measles PCR from a skin biopsy d. Rash in a patient nonimmune to measles

129. D Rash in a patient nonimmune to measles Rationale: A specific diagnosis of measles usually can be made on the basisof clinical presentation alone because of the characteristic prodrome (cough, coryza, and conjunctivitis with or without Koplik spots) and febrile exanthem. Detection of measles virus or antigens in clinical specimens or tissue can establish a recent infection with the measles virus. Viral isolation is possible from respiratory secretions, blood, urine, and, in special circumstances, skin biopsy. Serologic testing may be used to confirm the clinical diagnosis ofmeasles or to assess immunity. The presence of measles IgM antibody by enzyme immunoassay (EIA) confirms a diagnosis of ecent measles. Measles-specific IgM antibodies my not be present until 72 hours after the onset of the rash, however. Seroconversion or a fourfold increase in measles IgG antibody titer by EIA, hemagglutination inhibition (HAI), or neutralizing antibodies can also be used to confirm a ecent measles infection.

13. The IP has worked with the Preoperative Services manager to plan and implement practices to help decrease the bioburden of microorganisms on patients' skin prior to planned surgeries. Which of the following might be included in their plan? 1) Active surveillance culturing for epidemiologically significant organisms 2) Preoperative showering using antimicrobial soap 3) Preoperative antibiotics given prior to the "cut time" 4) Treatment of remote site infections prior to surgery a. 1, 2, 3 b. 1, 2, 4 c. 2, 3, 4 d. 1, 3, 4

13. B 1, 2, 4 Rationale: Prevention strategies to reduce SSIs include:• Administer antimicrobial prophylaxis in accordance with evidence-based standards and guidelines (within 1 hour prior to incision, 2 hours for vancomycin and fluooquinolones).• Identify and treat remote infections before elective operation.Preoperative antibiotics are not given to reduce the microorganism bioburden prior to surgery. The following are examples of activities that may reduce the bioload of microorganisms on patients' skin:• Washing from clean to less-clean areas using clean washcloths to prevent cross contamination• Preoperative showering using antimicrobial soap• Washing with antimicrobial soaps, such as chlorhexidine gluconate, to reduce carriage of resistant organisms, such as MRSA• Active surveillance culturing for epidemiologically significant oganisms based on the facility's epidemiology• Encouraging or assisting patients in maintaining good oral hygiene and caring for the mouth to reduce the risk of mucositis in immunosuppressed persons• Encouraging good genital-area cleansing• Good hand hygiene practices using soap and water or alcohol-based hand rubs, as appropriate• Treatment of remote site infections prior to surgery• Additional personal risk-reduction strategies, include smoking cessation or weight loss if possible and appropriate

130. Which of the following statements is true regarding the evaluation of an educational program? a. The evaluation should only be done at the end of the program b. Evaluation measurements must be consistent with the objectives of the program c. The evaluation always includes one-on-one interviews to assess the ability of the individual learner to perform d. The evaluation should not be used to monitor the behavior change—this is the role of the presenter

130. B Evaluation measurements must be consistent with the objectives of the program Rationale: The educator may use evaluation at different points within the program development process using a variety of methods. Whatever evaluation methodology is used, evaluation measurements must be consistent with the objectives established for the educational program. Data must be gathered, tabulated, and analyzed to assess impact and make recommendations for curriculum revision before the next presentation.

131. Which of the following studies is experimental rather than observational? a. Cohort b. Clinical trial c. Case-control d. Cross-sectional

131. B Clinical trial Rationale: Experimental studies are prospective studies designed to compare outcomes in individuals who are assigned to an experimental (intervention) or control (placebo or standard care) group. The intervention may be a procedure, drug, or other treatment, and the comparison group usually receives a placebo, the previously accepted treatment, or, if appropriate, no treatment. The two major types of experimental studies are randomized clinical and community trials.

132. Another name for "flash terilization" is: a. Immediate-use b. High-level disinfection c. Low-level disinfection d. Ethylene oxide (ETO) sterilization

132. A Immediate-use Rationale: Flash sterilization (flashing), a form of point-of-use processing, occurs immediately before use and close to the patent care area, where it will be used.

133. Because of the potential for rapid spread, one confirmed case o this disease is considered an urgent public health situation, and the IP should immediately report suspected and confirmed case to the health department: a. Chickenpox b. Influenza c. Measles d. Legionnaires' disease

133. C Measles Rationale: Measles (rubeola, 7-day measles) is a distinct clinical syndrome with characteristic prodrome respiratory tract symptoms (cough, coryza, and conjunctivitis), followed by a febrile exanthem and a recovery period that includes a persistent cough for many weeks. Measles occurs throughout the world. It is one of the most highly contagious infectious diseases in humans.

134. Maximal barrier precautions for central line insertion include: 1) Sterile surgical gown and gloves 2) Closing the door to the room 3) Nonsterile gown and gloves 4) Mask, cap, sterile drape (head to toe) a. 1, 4 b. 2, 3 c. 1, 3 d. 2, 4

134. A 1, 4 Rationale: Use maximal sterile barrier precautions, including the use of a cap, mask, sterile gown, sterile gloves, and a sterile full body drape for the insertion of central venous catheters, peripherally inserted central venous catheters, or guidewire exchanges.

135. The new IP for a LTCF assesses adherence to the facility's hand hygiene policies. In reporting her findings, she includes one o the following: a. The number of hand hygiene episodes performed by personnel divided by the volume of soap used in the facility b. The number of hand hygiene episodes performed by personnel divided by the number of patient days times 1,000 c. The number of hand hygiene episodes performed by personnel divided by the volume of alcohol-based hand rub d. The number of hand hygiene episodes performed by personnel divided by the number of hand hygiene opportunities by ward or service

135. D The number of hand hygiene episodes performed by personnel divided by the number of hand hygiene opportunities by ward or service Rationale: The CDC guideline and The Joint Commission require that HCP adherence to recommended hand hygiene policies be monitored and that HCP be provided with information about their performance. Acceptable methods for measuring hand hygiene adherence include:• Periodically conduct an observational study to determine the rate of adherence (number of hand hygiene episodes performed/number of hand hygiene opportunities) by ward or service. In addition to monitoring the rate of adherence, facilities may also assess the quality of hand hygiene adherence (time spent per hand hygiene episode, whether soap was used, etc.)• Monitor the volume of specific hand hygiene products (e.g., soap, hand rub, hand lotion) used per 1,000 patient days• Monitor adherence to artificial fingernail polici

63. An orthopedic surgeon is requesting that all "dirty" cases be scheduled for the end of the day. The IP's response is to: a. Ask the Director of Surgical Services to comply with his request b. Explain to the surgeon that operating rooms are cleaned after each case and that there is no benefit o scheduling procedures at the end of the day c. Provide the surgeon with the cleaning policy and ignore his request d. Request a meeting with the Environmental Services director to discuss

63. B Explain to the surgeon that operating rooms are cleaned after each case and that there is no benefit o scheduling procedures at the end of the day

. Which of the following employee infections would require that the healthcare worker be restricted from patient contact? a. Sinus infection being treated by antibiotic b. Small, painful vesicular lesion on the fingertip c. Shingles, which has been treated with an antiviral for past 4 days d. Dry, crusted lesion on right arm with no new drainage

14. B Small, painful vesicular lesion on the fingertip Rationale: Herpetic whitlow is a skin infection of the fingers, mot commonly found on the tips of the thumb and index finge. Herpetic whitlow is caused by the herpes simplex virus. Herpetic whitlow is common among HCP who come into contact with the virus regularly and in children who have the virus and suck their fingers or thum. Recommendations from the Advisory Committee on Immunization Practices (ACIP) include restricting HCP with herpetic whitlow from patient contact and contact with the patient's environment until the lesions have healed.

16. Which part of a published research study contains a brief summary of the entire research process with interpretation of data, conclusions from the data, limitations of the study and recommendations for further research? a. Introduction b. Methods c. Results d. Discussion

16. D Discussion Rationale: The discussion section of a research study reviews, interprets, and evaluates the results of the study. This section usually lists the hypothesis or hypotheses and reports whether the results supported or contradicted the hypothesis. Similarities and differences between the current findings andfindings of pevious research are reviewed. Limitations of the current study are discussed and suggestions are made on improving the research design. The discussion section usually ends with recommendations for future research.

17. An IP conducts an audit of the kitchen at an acute care facility. Which of the following would be a concern? a. Food servers wearing a plain wedding band b. Vegetables stored below raw meat in the refrigerator c. Nonabsorbent cutting boards d. Food stored 6 inches above floor

17. B Vegetables stored below raw meat in the refrigerator Rationale: Improper storage or environmental sanitation may introduce contamination or allow low microbial load to proliferate if not kept at appropriate temperatures. Food storage must be done in a systematic manner to ensure that all food safety requirements are met. Food must be protected from cross-contamination by separating raw animal foods during storage, preparation, holding, and display from raw ready-to-eat food, including fruits and vegetables, as well as cooked ready-to-eat foods per the FDA Food Code 3-302.11.

18. As the sample size increases, what tends to happen to the 95 percent confidence interval?a. The margin of error increases b. They become more precise c. They become wider d. They become less precise

18. B They become more precise Rationale: Larger sample sizes generate narrower and more precise confidence intervals. There is an inverse square root relationship between confidence intervals and sample sizes. To cut the margin of error in half, the sample size needs to be quadrupled, approximately.

19. Which of the following is not an Occupational Safety and Health Administration (OSHA) requirement for healthcare linens? a. Employers must launder all workers' personal protective garments or uniforms that are contaminated with blood or other potentially infectious materials (OPIM) b. Contaminated textiles should be bagged or otherwise contained at their point of use c. In the laundry facility, positive pressure should be maintained in the area where contaminated textiles are received d. Wet contaminated laundry must be placed in leakproof and color-coded or labeled containers at the location where it was used

19. C In the laundry facility, positive pressure should be maintained in the area where contaminated textiles are received Rationale: Laundry facilities must maintain negative pressure in the area where contaminated textiles are received compared with the clean areas of the facility.

41. Which would be the most objective method to determine the need for education about hand washing for a group of employees on a unit? a. Observational studies b. Personal interviews c. Focus group discussions d. Test development

41. A Observational studies Rationale: Methods that can be used to determine educational needs of the learner population include the following:• Learner self-assessment: The learner develops a self-achievement model and compares the current situation to the standard.• Focus group discussion: Learning needs are assessed in small groups with members assisting each other to clarify needs.• Interest-finder sureys: These are data-gathering tools, such as checklists or questionnaires.• Test development: Tests can be used as diagnostic tools to identify areas of learning deficiencies• Personal interviews: The educator consults with random or selected individuals to determine learning needs.• Job analysis and performance reviews: These methods provide specific, pecise information about work and performance.• Observational studies: Direct observation of personnel working can be performed by quality management analysts or IPs (e.g., hand-washing study in critical care units).• Review of internal reports: Incident reports, occupational injury and illness reports, and performance improvement studies can be reviewed to determine specific learning needs of healthcae providers

20. The microbiology lab calls the IP with a CSF Gram stain result. From an infection prevention and control standpoint, the most concerning result would be: a. Gram-negative rods b. Gram-positive cocci in pairs c. Acid-fast bacilli d. Gram-negative diplococci

20. D Gram-negative diplococci Rationale: Growth of oxidase-positive colonies and Gram-negative diplococci with the right clinical context provides a presumptive identification of N. meningitidis.

1. The reservoir of microbes of pathogens present in potable water and its delivery network include: 1) Staphlococci 2) Pseudomonas spp. 3) Nontuberculous Mycobacteria 4) Arthrobacter spp. a. 1, 2 b. 2, 3 c. 1, 4 d. 2, 4

21. B 2, 3 Rationale: Disruption of water utility systems during construction or renovation can disturb the biofilm present in water delivery pipes and pose a threat to patients. Microbes present in potable water and its delivery network include gram-negative bacteria, e.g., Legionellae and Pseudomonas spp, nontuberculous Mycobacteria, protozoa, and fungi.

23. A patient is admitted with watery diarrhea and abdominal cramps. The food history interview reveals that the patient consumed raw oysters recently. Which of the following organisms is the most likely cause? a. Yersinia b. Clostridium difficil c. Vibrio d. Campylobacter

23. C Vibrio Rationale: Vibrio gastroenteritis is caused by eating undercooked or raw seafood, such as fish and shellfish

24. A patient with a history of a cough greater than 3 weeks, night sweats, weight loss and a chest x-ray "suspicious for tuberculosis (TB)" is scheduled for a bronchoscopy. The procedure should be performed in which setting? a. The Operating Room under positive pressure b. Radiology c. An airborne infection isolation room with negative airflow d. Procedure room in Endoscopy

24. C An airborne infection isolation room with negative airflow Rationale: When TB is suspected, a bronchoscopy should be performed only if medically necessary. The procedure should only be performed in a room that meets the ventilation requirements for an AIIR (negative directional airflw, a minimum of 12 air exchanges per hour, and direct exhaust to the outside more than 25 feet from an air intake or discharge through a high-efficieny particulate air [HEPA] filtation system).

25. A patient tells his nurse that he thinks his apartment is infested with bed bugs. Upon examination, the nurse finds bites on the patient's legs. The nurse contacts you for direction. Your response to the nurse is: a. Assess the hospital room for mosquitoes b. Place the patient's belongings is a plastic bag and tie securely c. Request that the belongings be incinerated d. This might be a rash related to a new antimicrobial that the patient is taking

25. B Place the patient's belongings in a plastic bag and tie securely Rationale: If bed bugs are suspected, the patient must be examined and his or her personal belongings should be placed in plastic bags that are tied securely closed or in other sealed containers. Environmental clutter should be removed and the area vacuumed, preferably using a HEPA-filtered vacuum.

26. A p value expressed as p <0.01 indicates: a. The possibility of these results occurring by chance alone is very small, so therefore, the result is not significant b. The possibility of these results occurring by chance alone is less than 1 in 100 and, therefore, significant enough to prove causality c. The null hypothesis should be rejected and the alternative hypothesis should be accepted d. The null hypothesis should be accepted

26. C The null hypothesis should be rejected and the alternative hypothesis should be accepted

27. Which of the following must be documented when using soaking solutions for cleaning instruments? 1) Monitoring the temperature of the solution 2) Monitoring the room temperature 3) Monitoring the humidity of the room 4) The time the instruments soaked a. 1, 2 b. 2, 4 c. 3, 4 d. 1, 4

27. D 1, 4 Rationale: The temperature of the soaking solution should be monitored and documented to ensure that the temperature of the cleaning solution meets the requirement of the cleaning solution's written IFU. The time the instruments are soaked should also be recorded.

28. The Director of Infection Prevention and Control and her infection prevention staff have decided to develop a clinical practice guideline to decrease the rate of CLABSIs in their facility. The goal is to reduce variation in practice and improve clinical outcomes. Of the choices below, what is important to remember when developing this guideline? a. The guideline should be no more than two pages in length to ensure compliance b. The guideline should categorize and compare data against a unit of measurement c. The guideline should identify, summarize, and evaluate the highest-quality evidence and most current data in the literature d. The guideline should identify how process specifications change over time

28. C The guideline should identify, summarize, and evaluate the highest-quality evidence and most current data in the literature Rationale: Clinical practice guidelines are evidenced-based standards, such as algorithms and consensus statements, that address reducing variation in practice and improving clinical outcomes.

30. A hospital has admitted an immunocompetent patient with localized herpes zoster. During unit rounds, the IP notes a sign on the door that says "No pregnant women." How should the IP best respond to this? a. Leave the sign on the door because the patient poses a high risk of disease transmission to pregnant women b. Remove the sign from the door but ask the charge nurse to not assign the patient to any pregnant HCP and to prevent pregnant visitors from entering the room c. Remove the sign from the door and place the patient on both Contact and Airborne Precautions d. Remove the sign from the door and ensure that all lesions are completely covered; Standard Precautions are sufficient o prevent the spread of the virus

30. D Remove the sign from the door and ensure that all lesions are completely covered; Standard Precautions are sufficient o prevent the spread of the virus Rationale: According to CDC recommendations, following Standard Precautions and completely covering lesions are sufficient o prevent transmission of localized zoster to susceptible visitors and personnel. If a woman has a history of varicella-zoster infection or vaccination, her antibodies will protect her fetus from infection, so there is no risk for her to enter the room or care for the patient. Infection with zoster virus during pregnancy can be harmful to the fetus; however, all HCP should be immune to zoster through history of illness or through immunization upon employment. If unvaccinated or susceptible HCP at risk for severe disease and for whom vaccination is contraindicated (e.g., pregnant HCP) are exposed, the CDC recommends that those personnel receive VZIG after exposure.

32. Using Improved Hydrogen Peroxide offers many advantages. Those advantages are: 1) Rapid action 2) Low toxicity 3) Effective against spores 4) Low cost a. 1, 2 b. 2, 3 c. 2, 4 d. 1, 4

32. A 1, 2 Rationale: A major advantage of improved hydrogen peroxide is its rapid action, high effectiveness, and low toxicity. A disadvantage is it is more expensive than a quaternary ammonium compound in ready-to-use containers. Improved hydrogen peroxide is considered safe for humans and equipment, and benign for the environment. In fact, improved hydrogen peroxide has the lowest EPA toxicity category based on its oral, inhalation, and dermal toxicity, which means that it is practically nontoxic and is not an irritant.

33. Scabies is transmitted through: a. Sharing combs and brushes b. Handling books or magazines after a person infested with scabies c. Direct contact with infested skin d. Scabies mites crawling from person to person

33. C Direct contact with infested skin Rationale: Transfer of parasites commonly occurs through prolonged direct contact with infested skin and also during sexual contact. Transfer from undergarments and bedclothes occurs only if these have been contaminated by infested persons immediately beforehand. People with crusted scabies syndrome are highly contagious because of the large number of mites present in the exfoliating scales.

34. Between cases in the operating room, the floors must cleaned and disinfected as follows: a. Floors are only cleaned at the end of the day unless organic debris is present b. The entire floor must be flooded and cleaned between each case c. Spot clean the floor only if visible blood/body fluid. d. It is only necessary to clean a 3 to 4 foot perimeter around the table, unless a wider contamination area is identified

34. D It is only necessary to clean a 3 to 4 foot perimeter around the table, unless a wider contamination area is identifie Rationale: Floors in the operating room must be cleaned and disinfected after each case. Reusable string or microfiber mops my be used in between cases and should be changed after each use. If a cotton mop and bucket system is used, a clean mop head and fresh disinfectant must be used for each case. It is only necessary to clean a 3 to 4 foot perimeter around the operative table after each case unless a wider perimeter of contamination is identified.

35. A patient with a confirmed diagnosis of varicella (chickenpox) is seen in a busy ED. The staff at the registration desk immediately placed a mask on the patient until he could be moved to a negative airflow room in the ED, where he then removed his mask. A pregnant environmental services (EVS) employee, who is nonimmune to varicella, enters the room before an isolation sign is posted. She spends 6 minutes in the room with the patient, who is no longer wearing a mask. Which of the following is the most appropriate postexposure response? a. Give the EVS employee VZIG and place off work from day 10 through day 28 b. Administer the varicella vaccine immediately and place employee off work from day 10 through day 21 c. Have the employee wear a mask from day 10 through day 21 after the exposure and watch for signs and symptoms of disease d. Give the EVS employee the vaccine and VZIG and keep off work from day 10 through day 28 after the exposure

35. A Give the EVS employee VZIG and place off work from day 10 through day 28 Rationale: According to the recommended immunization practices by the U.S. Public Health Service's ACIP, varicella vaccine should not be administered to pregnant women because the possible effects on fetal development are unknown. The EVS employee should receive the VZIG, and she should be excluded from duty for 28 days (see Table PE3-1).

36. Training on the use of PPE for environmental service employees must include: 1) When to wear PPE 2) The limitations of each type of PPE 3) Maintenance of PPE 4) The cost of PPE a. 1, 2, 3 b. 2, 3, 4 c. 1, 3, 4 d. 1, 2, 4

36. A 1, 2, 3 Rationale: The PPE training program must include the following:1. When PPE is worn2. What PPE to wear3. How to don, remove, adjust, and wear each type of PPE4. Limitations of each type of PPE5. Care, maintenance, useful life, and storage or disposal of PPE6. Written certification indicating

37. Bed bugs can survive without feeding for: a. 48 hours b. 24 days c. 6 month d. 1 year

37. D 1 year Rationale: Immature bed bugs may live for several months without feeding, while adults may survive as long as one year without a meal. Under normal conditions, adult bed bugs will live for about ten to eleven months.

38. A culture of a patient's dialysis access site grew Gram-positive cocci. One of the factors contributing to this type of infection is: a. Contamination of the water used in dialysis b. Skin colonization with S. aureus at the access site c. Contamination of the antiseptic used to prep the access site d. Seeding of the access site by remote sites of infection

38. B Skin colonization with S. aureus at the access site Rationale: Skin colonization with S. aureus at the access site has been significantly associated with S. aureus access site infections. Persistence of S. aureus after skin preparation has been shown to be significantly higher in patients with poor hygiene. The importance of personal hygiene and its possible relation to access site infections should be emphasized to patients. All patients should wash their access site with soap and water daily and before hemodialysis. Patients should also be instructed to ensure that all staff accessing the access site are preparing the skin appropriately prior to cannulation and wearing a mask for all access connections.

42. The IP is selecting a process measure in order to monitor and evaluate quality of care. An example of a process measure is: a. SSI after a hip replacement b. Conjunctivitis c. Hepatitis B immunity rates d. Tuberculin skin test (TST) conversions

42. C Hepatitis B immunity rates Rationale: Process measures are commonly used to evaluate compliance with desired care or support practices or to monitor variation in these practices. They may also be used when the outcome to be measured is rare or difficult to measure (e.g., infections after endoscopy) or when there is difficulty acquiring the data (e.g., contacting the discharged and relocated patient after surgery). Process measures are also helpful in evaluating the effectiveness of an educational effort as a measure of behavior (e.g., compliance with aseptic technique for dressing change) or performance of basic infection prevention procedures, such as hand hygiene. Hepatitis B immunity rates are an example of a process indicator. The other answers are examples of outcome indicators.

43. IPs should have a thorough understanding of the following when choosing a cleaning product: 1) Differences among types and uses of disinfectants 2) Manufacturing companies that produce and sell disinfectants 3) Definitions of the terms cleaning, sanitization, disinfection and sanitizer 4) Manufacturer's recommendations for use, including dilution and contact time a. 1, 2, 3 b. 2, 3, 4 c. 1, 3, 4 d. 1, 2, 4

43. C 1, 3, 4 Rationale: Environmental services managers, IPs, and other individuals responsible for selecting, purchasing, and/or educating others on the use of cleaning products should have a thorough understanding of the differences and uses of these chemicals. Specifically, these individuals should know:• Definitions of the terms cleaning, sanitization, disinfection, and sterilization• Definitions of soa, antiseptic, disinfectant, sterilant, and sanitizer• Differences among the types and uses of antiseptics• Differences among the types and uses of disinfectants, including sporicidal disinfectants• Differences among a disinfectant, a disinfectant-detergent, and a cleaning agent containing no antimicrobial properties• Manufacturer's specifiction for use, including dilution and contact time• Difference between products registered with the EPA and the FDA

44. The Surgical Scheduling Department called the IP stating that a patient scheduled for surgery in on December 24 will need to reschedule his procedure due to an exposure to scarlet fever on December 3. He has no signs or symptoms. How should the IP respond? a. The incubation period for scarlet fever is usually 1 to 7 days; he is outside the incubation period and will be able to have his surgery on the scheduled date b. The procedure should be rescheduled c. Request an order from the surgeon for antibiotics to treat the possible exposure d. Place the patient in Isolation Precautions upon admission

44. A The incubation period for scarlet fever is usually 1 to 7 days; he is outside the incubation period and will be able to have his surgery on the scheduled date Rationale: Group A Streptococcus (GAS) is one of the most frequent and important pathogens in humans. Spread of GAS occurs by direct person-to-person contact. Symptoms of scarlet fever typically appear 3 days after exposure to another person with the illness, though this incubation period can be anywhere from 1 to 7 days. When scarlet fever occurs, it is usually the result of pharyngeal infection, but it can occur in the setting of GAS skin infection or sepsis. It is characterized by a diffuse red rash that typically appears on the second day of illness, lends a sandpaper texture to the skin, and fades over the course of a week. It is followed by desquamation of the skin. The patient is outside the incubation period and will be able to have his surgery on the scheduled date. He should also be instructed to call back if he develops a fever or other signs/symptoms of an illness prior to the surgery date.

45. Which of the following helps to prevent the aerosolization of spores during dusting? 1) A damp paper towel 2) A chemically treated cloth or dust mop 3) A microfiber cloth or dust mop 4) A dry cotton wash cloth a. 1, 2 b. 2, 3 c. 1, 4 d. 2, 4

45. B 2, 3 Rationale: Dust contains fungal spores. To capture dust without aerosolizing spores, dusting should be performed using a cloth or dust mop that is chemically treated or made of microfibe. Dusters should never be shaken.

46. A physician would like to use a new screening test for methicillin-resistant Staphylococcus aureus (MRSA), which is highly prevalent in the hospital population. The screening test has a sensitivity of 98 percent and a specificity of 58 percent. Which of the following conclusions is most accurate about this screening test? 1) The test will be very effective in correctly identifying people who have MRSA 2) The test will be very effective in correctly identifying people who do not have MRSA 3) The test will yield very few false-negative results 4) The test will yield very few false-positive results a. 1, 3 b. 1, 4 c. 2, 3 d. 2, 4

46. A 1, 3 Rationale: A high sensitivity means that most people who actually have the disease will have a positive test result; this also means that few people who have the disease will have a negative test result, so the number of false negatives will be low

54. A 70-year-old patient is admitted with symptoms of rapid neurological degeneration. The physician orders the following diagnostic tests: 14 - 3 - 3 spinal fluid assay, an electroencephalogram (EEG), and magnetic resonance imaging (MRI). These tests, if positive, may indicate a possible diagnosis of: a. A brain tumor b. Viral encephalopathy c. Multiple sclerosis d. Creutzfeldt-Jakob disease

54. D Creutzfeldt-Jakob disease

.47. The risk of infection or adverse reactions in the Dialysis Unit can be reduced by which of the following interventions? 1) Test patients and staff for MRSA colonization and treat as needed 2) Adhere to aseptic technique during all dialysis procedures 3) Have patient cleanse the access site with soap and water daily and prior to dialysis 4) Have patients take a prophylactic antibiotic prior to dialysis a. 1, 3 b. 2, 3 c. 1, 2 d. 1, 3

47. B 2, 3 Rationale: Most common types of dialysis-associated infections include access site infection, bacteremia, and peritonitis. The risk of infection or adverse reactions in the dialysis unit can be reduced by strict adherence to aseptic technique during all dialysis procedures. Skin colonization with S. aureus at the access site has been significantly associated with S. aureus access site infections. Persistence of S. aureus after skin preparation has been shown to be significantly higher in patients with poor hygiene. Therefore, the importance of personal hygiene and its possible relation to access site infections should be emphasized. It is recommended that all patients be taught to wash their access site with soap and water daily and before hemodialysis.

48. An IP is interested in evaluating whether her educational program on utilizing PPE has led to applied changes in practice. What is the best method of determining this? a. Use a pretest and posttest evaluation to see how knowledge of PPE use has changed after the educational program b. Use formative evaluation of the program so she can make necessary changes before it is implemented c. Use exit questionnaires to find out whether participantsintend to implement their new knowledge in a practice situation d. Ask the supervisor to collect data on direct observation of practice

48. D Ask the supervisor to collect data on direct observation of practice Rationale: Educators may use evaluation at different points within the program development process. Formative evaluation is conducted during the planning of the educational session to provide immediate feedback and to allow appropriate changes to be made. Data collected by pretest and posttest before and after an intervention are used to measure change in individual or group understanding of the content but will not provide information on application of knowledge in a practice setting. Exit questionnaires are used to gather information about the overall success of the program to ask for feedback on all aspects of the course, including objectives, the presenter, the quality of teaching aids, and the learning environment. Direct observation of practice would provide the best method to assess whether employees are applying the learning objectives on the job.

15. You are participating in a team to select and evaluate novel environmental disinfectant products. When preparing a presentation to the team on the advantages of ultra violet (UV) irradiation, it is important to include information on: 1) Patients may remain in the room with the device operating 2) Decontamination can be achieved in 2.5 hours 3) HVAC system does not need to be disabled 4) Ability to achieve reductions in vegetative bacteria a. 1, 4 b. 2, 4 c. 3, 4 d. 1, 3

5. C 3, 4 Rationale: UV irradiation has been used for the control of pathogenic microorganisms in a variety of applications, such as control of Legionellosis, as well as disinfection of air, surfaces, and instruments. UV irradiation is effective substantially reducing levels of vegetative bacteria. All patients and staff must be removed from the room prior to decontamination. Rooms can be rapidly decontaminated of vegetative bacteria in 15 to 20 minutes. All patients and staff must be removed from the room prior to decontamination

50. A patient who was recently vaccinated against smallpox has been admitted to a healthcare facility for a condition unrelated to the immunization. The charge nurse would like to know what type of precautions this patient needs. a. Standard Precautions b. Contact Precautions c. Droplet Precautions d. Contact and Airborne Precautions

50. A Standard Precautions Rationale: Vaccinia virus is the poxvirus that is used to vaccinate against smallpox. The vaccine is a live vaccine that is administered using a bifurcated needle; a positive vaccine reaction results in a pustule at the vaccine site that lasts for approximately 30 days. Live virus can be transmitted through direct contact with the pustule or the scab that grows over it, but in a healthy person the pustule is limited to the vaccine site and is not widespread. The pustule should be covered with a bandage to prevent the patient from self-inoculating other sites of his own body. HCP should use Standard Precautions in this case, including wearing gloves to change the vaccine site dressing. Conditions directly related to immunization including eczema vaccinatum and generalized vaccinia warrant Contact Precautions until the immunization site has dried out and the scab has separated from the skin.

51. The organization that would most likely survey a U.S.-based hospital's blood bank for compliance would be: a. Occupational Safety and Health Administration (OSHA) b. Food and Drug Administration (FDA) c. Environmental Protection Agency (EPA) d. Centers for Disease Control and Prevention (CDC)

51. B Food and Drug Administration (FDA) Rationale: The FDA is responsible for the safety of the nation's blood supply. The FDA has specific standards for collection, testing, and distribution of blood, as well as disposal of contaminated or untested blood. These standards apply to all facilities that have blood-banking operations.

C. difficil spores can survive in the environment for: a. 24 months b. 3 months c. 5 months d. 12 months

52. C 5 months Rationale: CDI has emerged as a major pathogen of healthcare-associated infections. Multiple outbreaks caused by strains with high mortality rates have been reported. The organism produces spores in feces. Surfaces contaminated with feces become a potential reservoir of spores, which can survive in the environment for 5 months.

53. Which of the following describes aerobic bacteria? a. Do not require oxygen to grow and are harmed by its presence b. Require oxygen to grow and survive c. Can grow with or without oxygen d. Do not require oxygen for growth but tolerate the presence of it

53. B Require oxygen to grow and survive Rationale: Oxygen is a universal component of cells. Prokaryotes display a wide range of responses to oxygen. Aerobic bacteria require oxygen to grow and survive.

55. A paramedic has been exposed to a patient's blood. The patient is Hepatitis B e Antigen (HBeAg) positive and the paramedic is Anti-HBc and Anti-HBs positive. What is the risk of the paramedic acquiring Hepatitis B infection from the source patient? a. There is no risk of infection for the paramedic because he was previously infected and is now immune b. There is no risk of infection for the paramedic because the patient is not infected c. There is no risk of infection for the paramedic because he is immune due to vaccination d. There is a high risk of infection for the paramedic

55. A There is no risk of infection for the paramedic because he was previously infected and is now immune Rationale: The patient is infected and can transmit Hepatitis B, but the paramedic has been previously infected and is now immune to Hepatitis B. Antibody to the Hepatitis B core antigen is made in response to actual infection with the virus and antibody to the HBsAg is made after the virus is cleared from the patient. Anti-HBs prevent the person from becoming re-infected.

revisit

56. A Total number of patients screened Rationale: The numerator is the actual number of patients screened. Basic Formula for All Types of Rates• Rate = x/y × k Where: • x = The numerator, which equals the number of times the event (e.g., infections) has occurred during a specified time interval. • y = The denominator, which equals a population (e.g., number of patients at risk) from which those experiencing the event were derived during the same time interval. • k = A constant used to transform the result of division into a uniform quantity so that it can be compared with other, similar quantities. A whole number (fractions are inconvenient) such as 100, 1,000, 10,000, or 100,000 is usually used (selection of k is usually made so that the smallest rate calculated has at least one digit to the left of the decimal point) or is determined by accepted practice (the magnitude of numerator compared with denominator).

57. Subjects are said to be randomly assigned when: a. They are assigned to experimental and control groups from a sample representative of the larger experimental group b. They have an equal chance of being assigned to either the experimental or the control group c. They are assigned to experimental and control groups so that the groups differ on a critical variable d. Both the researcher and the subject are blinded as to know whether the subject is in the control group or the experimental group

57. B They have an equal chance of being assigned to either the experimental or the control group Rationale: Random assignment is a procedure used in experimental studies to create multiple study groups that include subjects with similar characteristics so that the groups are equivalent at the beginning of the study. Participants are assigned to an experimental treatment or program at random so that each individual has an equal chance of being assigned to either group.

58. Privacy curtains are high-touch items that should be changed and cleaned: 1) On a routine schedule and when soiled 2) During construction 3) According to manufacturer's instructions 4) After a patient on Contact Isolation is discharged or transferred a. 1, 2 b. 2, 3 c. 3, 4 d. 1, 4

58. D 1, 4 Rationale: Privacy curtains are considered high-touch items and are to be changed and cleaned on a routine schedule and whenever soiled. It is also recommended that they be changed after a patient on Contact Isolation is either transferred or discharged.

59. Examples of high-level disinfectants are: 1) Glutaraldehyde 2) Hydrogen peroxide 3) Enzymatics 4) 70 percent ethyl alcohol a. 1, 3 b. 1, 2 c. 2, 3 d. 1, 4

59. B 1, 2 Rationale: Semi-critical items minimally require high-level disinfection using chemical disinfectants. Glutaraldehyde, hydrogen peroxide, orthophthaladehyde, improved hydrogen peroxide, peracetic acid with hydrogen peroxide, and chlorine-based products are approved by the U.S. Food and Drug Administration and are dependable high-level disinfectants provided the factors influencing germicidal pocedures are met.

60. A patient with MRSA infection has been admitted and is in need of physical therapy. The physical therapist would like the patient to ambulate in the hall. The IP should recommend: a. The patient should not ambulate in the hall b. The patient may ambulate but only outside of the hospital c. The patient may ambulate in the hallway if attendants wear appropriate PPE d. The patient may ambulate in the hallway if he washes his hands and wears a clean gown

60. D The patient may ambulate in the hallway if he washes his hands and wears a clean gown

61. The IP is a member of an interdisciplinary team that has been given administrative oversight for its planning, activity scheduling, and expenditures under $500. This type of team is best described as: a. Cross-functional b. Virtual c. Autonomous d. Self-managed

61. D Self-managed Rationale: Self-managed teams, also called self-directed teams, are small autonomous groups of employees who determine, plan, and manage their daily activities with little or no supervision. Self-managed teams offer cost savings and increased productivity.

62. Commercial third-party reprocessors are regulated by which government agency? a. Centers for Disease Control and Prevention (CDC) b. Environmental Protection Agency (EPA) c. Center for Medicaid/Medicare Services (CMS) d. Food and Drug Administration (FDA)

62. D Food and Drug Administration (FDA) Rationale: When considering reprocessing SUDs, hospitals are faced with the decision whether to contract with a third-party reprocessor or formulate an in-house plan. No matter the approach, the process must comply with the FDA regulation

65. During an inservice for new employees, the IP describes how Hepatitis B and human immunodeficieny virus (HIV) are transmitted. A major difference in the epidemiology of the two diseases is: a. Presence of the causative agent in body fluid b. The ability of the diseases to be transmitted during sexual intercourse c. The risk of transmission through needlestick exposures d. The potential for airborne transmission

65. C The risk of transmission through needlestick exposures Rationale: Infection with HBV is a well-recognized occupational risk for HCP. The risk of HBV infection is primarily related to the degree of contact with blood in the work place and also the HBeAg status of the source person. The risk of developing clinical hepatitis if the blood was positive for both HBsAg and HBeAg has been estimated at 22 to 31 percent. In contrast, the average risk of HIV transmission after a percutaneous exposure to HIV-infected blood has been estimated to be approximately 0.3 percent.

66. Which of the following are attributes of a culture of safety in a healthcare organization? 1) All HCP accept responsibility for safety 2) The organization prioritizes safety over financial an operational goals 3) Only personnel in leadership positions may report safety issues 4) The organization prioritizes identifying and reprimanding individuals who are responsible for accidents a. 1, 2 b. 2, 3 c. 3, 4 d. 1, 4

66. A 1, 2 Rationale: The NPSF outlines fie attributes of a safety culture that all healthcare organizations should strive to operationalize through implementation of string safety management systems:• All workers (including front-line staff, physicians, and administrators) accept responsibility for the safety of themselves, their coworkers, patients, and visitors• Safety has priority over financial and operational goals• The organization encourages and rewards the identification, communication, and resolution of safety issues• There are provisions for organizational learning from accidents• The organization allocates appropriate resources, structure, and accountability to maintain effective safety systems

67. Last year, a hospital identified 21 CLABSIs. Which type of chart would be most useful to provide feedback regarding the effectiveness of CLABSI reduction strategies? a. Control chart b. Pie chart c. Run chart d. Display of normal distribution and standard deviation

67. C Run chart Rationale: Two common methods used to measure and plot variation in the process of care include run charts and control charts. A run chart is the simplest of charts. It is a single line plotting observed data over time. A run chart can help identify upward and downward trends, and it can show a general picture of a process. A control chart also plots a single line of data over time. However, control charts include upper and lower control limit lines with a centerline. Control charts are more sensitive at detecting abnormalities than run charts but require at least 25 data points for reliability and validity. Run charts require at least 20 data points. With such a small number of observations in our data set, the run chart would be the most appropriate choice

68. Which of the following are allowed in an ICU or other unit with immunocompromised patients? a. Latex-free balloons b. Flowers in a vase c. Fresh fruit d. Fish tank

68. A Latex-free balloons Rationale: Infectious diseases cause significant morbidity and mortality in immunocompromised patients. Water is a reservoir for pathogenic microorganisms and can be a source for HAIs. As organizations recognize the importance of care delivery sites as therapeutic environments, they may consider installation of features such as fish tanks, decorative water fountains, water walls, or other water features. In balancing the risk of adding a potential reservoir of waterborne opportunistic pathogens, the CDC Guidelines for Environmental Infection Control in Health-Care Facilities recommend facilities avoid placing them in patient care areas. It is well established that both potted plants and fresh flwers carry microbial floa that are pathogenic for the immunocompromised host. Fresh fruits and vegetables also carry several species of Gram-negative rods as part of their natural floa. Latex-free balloons are a safe choice in healthcare facilities.

69. Which of the following educational tools is most effective to bridge the learning gap between theory and actual practice? a. Case study b. Lecture c. Computer-based training d. Game

69. A Case study Rationale: Case studies can be used as a training method to help bridge the learning gap between theory and actual practice. The method builds on a variety of learner skills: analytical, critical, and interactive. Learners explore multiple solutions and enhance creativity and problem-solving approaches often using a discussion-based format.

70. The incidence of VRE in the Burn Unit last quarter was 3.0, and the incidence of VRE in the Bone Marrow Transplant Unit last quarter was 1.0. What was the relative risk (RR) of new cases of VRE in the Burn Unit compared to the Bone Marrow Transplant Unit? a. RR = 3.0 b. RR = 0.33 c. RR = 4.0 d. RR = 2.0

70. A RR = 3.0 Rationale: RR is the incidence in the exposed group divided by the incidence in the unexposed group. This case is simply comparing the incidence in two units. Because the question asks for the relative risk in the Burn Unit as compared to the Bone Marrow Transplant Unit, the Burn Unit should be considered to be the exposed group. The RR is the incidence in the Burn Unit divided by the incidence in the Bone Marrow Transplant Unit, which is 3.0/1.0. This is equal to 3.0. Thus patients in the burn unit had 3 times higher risk of newly acquired VRE than patients in the bone marrow transplant unit.

86. What species of the streptococci can be transferred to an infant during delivery? a. Streptococcus pyogeses b. Xanthomonas c. Streptococcus agalactiae d. Enterococcus faecalis

86. C Streptococcus agalactiae Rationale: Asymptomatic carriage of Streptococcus agalactiae or group B Streptococcus (group B strep) in gastrointestinal and genital tracts is common. Intrapartum transmission via ascending spread from vagina occurs. Neurologic sequelae include sight or hearing loss and cerebral palsy. Death occurs in 5 percent of infants.

71. There has been a major bioterrorism event in the area, as is evidenced by a large number of patients being admitted to local healthcare facilities with fever >101.1° F and respiratory symptoms that are progressing rapidly. Health officials ae currently unaware of the causative organism. What type of Isolation Precautions are warranted in this situation? a. Standard Precautions b. Droplet Precautions c. Airborne Precautions d. Contact and Airborne Precautions

71. C Airborne Precautions Rationale: During the beginning of an infectious disease disaster when the agent may not have been identified or when thee is not enough evidence regarding the disease transmission route, IPs need to base infection prevention decisions on syndromes and symptomatology. This is referred to as syndrome-based isolation/control measures. General guidelines include implementing Airborne Precautions if the patient is severely ill with rapidly progressing respiratory symptoms and an airborne spread disease is suspected (i.e., severe acute respiratory syndrome or avian influenza)

72. The IP has identified an increased incidence of catheter-related bloodstream infections in the NICU. The IP is preparing her report for presentation to the Infection Prevention Committee. Among the following actions listed, which should be included in the IP's report as the next step? a. Changing the type of catheter used b. Using a multidisciplinary approach to determine corrective actions c. Revising the NICU admission policy d. Revising the NICU staffing plan

72. B Using a multidisciplinary approach to determine corrective actions Rationale: Answers A, C, and D may be actions recommended following thorough review and discussion of the surveillance data, but they are not the next steps to take without firt seeking input on the appropriate corrective actions from the appropriate stakeholders.

73. You have been invited to tour a commercial third-party reprocessor that your organization is considering using to reprocess some equipment. As the IP, you should request to observe which of the following? 1) Observe the cleaning and decontamination process 2) Observe hand hygiene 3) Sterilizing load preparation process 4) New employee orientation a. 1, 2 b. 2, 4 c. 2, 3 d. 1, 3

73. D 1, 3 Rationale: Healthcare facilities considering contracting with a commercial third-party reprocessor have the responsibility of knowing that reprocessing an SUD presents no greater risk to their patients' health and safety. An on-site visit should be scheduled, with the opportunity to meet with personnel involved in the process, and a review of the company's policies. The visit should include an opportunity to observe the cleaning and decontamination, inspection and testing, and sterilization load preparation processes, as well as reviewing quality control records.

74. An example of a molecular testing methodology is: a. Gram stain b. Culture c. Polymerase chain reaction (PCR) d. Point-of-care testing

74. C Polymerase chain reaction (PCR) Rationale: Molecular testing methodologies have greatly enhanced the speed, specificity, and sensitivity of tests for clinically significant microbes. Examples of molecular testing methods are PCR, pulse field gel electophoresis, Western blot assay, enzyme linked immunoassays, and molecular genotypic assays.

75. There is a bioterrorism event in a city. Patients are presenting to the Emergency Room with blurred vision, descending symmetrical flacid paralysis, and respiratory failure. The most likely bioterrorism disease is: a. Botulism b. Pneumonic plague c. Viral hemorrhagic fever d. Smallpox

75. A Botulism Rationale: Botulism is a public health emergency. Botulism is a neuroparalytic illness caused by a toxin made by the bacterium C. botulinum. Symptoms of botulism include blurred vision, diplopia, dysarthria, dysphagia, symmetrical descending flacid paralysis and respiratory failure. Prompt diagnosis and early treatment of botulism are essential to minimize the number of affected persons and the severity of illness.

76. A healthcare facility requires TB screening for all employees at time of hire using two-step tuberculin skin test (TST) with purified protein derivative (PPD). A newly hired employee has provided documentation of one negative TST from 6 months prior. Which of the following actions should the IP recommend? a. No additional TSTs are needed b. Place one additional TST and screen for symptoms c. Place two TSTs 1 week apart d. Obtain a chest X-ray on the employee

76. B Place one additional TST and screen for symptoms Rationale: HCP with documentation of a previous negative TST within the prior 12 months should have a second TST placed and be screened for signs and symptoms of TB. The TST should be read in 48 to 72 hours. If negative, the employee is cleared for work. If the TST is positive, a chest x-ray should be performed.

77. Which of the following is an example of a continuous data set? 1) Body temperature measurements: 98.6°F, 97.4°F, 99.8°F, 99.9°F 2) Gender: male, male, female, male 3) Blood type: O+, A-, A+, AB+ 4) Body weight: 189, 144, 261, 113 a. 1, 2 b. 2, 3 c. 2, 4 d. 1, 4

77. D 1, 4 Rationale: Continuous data contain information that can be measured on a continuum or scale and can have numeric values between the minimum and maximum value (a continuum) (e.g., age; serum cholesterol level; temperature, such as 98.6°F, 98.7°F, and 98.8°F; infection rates); continuous data require the process of measuring, rather than counting, and may contain whole numbers, decimals, or percentages. Conversely, discrete data contain whole numbers and are mutually exclusive (e.g., infected or not infected, male or female, blood type).

78. Which of the following refers to the ability to evaluate, compare, analyze, critique, and synthesize information? a. Inductive thinking b. Critical thinking c. Transductive thinking d. Deductive thinking

78. B Critical thinking Rationale: Critical thinking is the identification and evaluation of evidence to guide decision-making. The list of core critical thinking skills includes observation, interpretation, analysis, inference, evaluation, explanation, and metacognition. Critical thinking is imperative when evaluating and interpreting research studies.

79. The greatest concern in an emergency situation involving a patient with nontraditional body piercing is: a. Bleeding from areas of high vascularity b. Hematogeneous spread of bacteria to other sites c. Not being familiar with the opening mechanism of the jewelry to be able to remove it d. Causing distress to the patient

79. C Not being familiar with the opening mechanism of the jewelry to be able to remove it Rationale: It is becoming more common for HCP to provide care for patients with body jewelry, and it is sometimes unclear whether removal of the jewelry is necessary. Removal of these items is sometimes required for radiological purposes and removal is not usually difficult if the patient is able to assist with the opening mechanism. In an emergency situation, attempts at removal may cause unnecessary trauma to the site. Many HCP are unaware of the procedures for removing body piercing. In a survey of 28 accident and emergency doctors, only six were able to accurately describe the opening mechanisms of all three commonly used types of jewelry.

80. A patient in your facility has an infection with Cryptosporidium parvum. Which disinfectant would you recommend for use on potentially contaminated patient equipment? a. Ethyl alcohol b. Six percent hypochlorite c. Ortho-phthalaldehyde d. Six percent hydrogen peroxide

80. D Six percent hydrogen peroxide Rationale: Cryptosporidium parvum are protozoa that are resistant to many disinfectants, including chlorine. The only disinfectant with known effectiveness against C. parvum at working concentrations is hydrogen peroxide.

81. The Infection Control Risk Assessment (ICRA) should include all of the following elements related to building design features except: a. Sewage systems to allow adequate sanitation of waste b. Location of special ventilation and filtration of heating, ventilation, air conditioning (HVAC) serving areas, such as ED waiting and intake areas c. Water systems to limit Legionella spp. and other waterborne opportunistic pathogens d. Air handling and ventilation for surgical services and airborne infection isolation (AII)

81. A Sewage systems to allow adequate sanitation of waste Rationale: Planning for new construction or major renovation requires early collaboration among IPs, epidemiologists, architects, engineers, and other stakeholders to ensure that design of specific structures facilitates desired infection prevention program practices. An essential first step in the planning process is ICRA, followed by interventions, monitoring, and continuous assessment and improvement at a broad, organizational program level and during operational projects. ICRA elements related to building design features include the following:• Numbers, location, and types of AII and protective environment (PE) rooms• Location of special ventilation and filtration of HVAC serving such areas as emergency department waiting and intake areas• Air handling and ventilation needs in surgical services, AII and PE rooms, laboratories, local exhaust systems for hazardous agents/chemicals, and other areas with special needs• Water systems to limit Legionella spp. and other waterborne opportunistic pathogens• Finishes and surfaces

82. An infection prevention consultant is reviewing projected expenses for the next 2 years. Although all of the following are necessary to support the consultancy, which is considered a capital expense? a. An increase in office rent over 2 years b. Personal liability insurance c. Building utilities in which the office is located d. Expanded computer hardware and software

82. D Expanded computer hardware and software Rationale: Capital expenses or expenditures are business expenses for fixed assets such as buildings or equipment. Operating expenses such as rent, utilities, and insurance are not considered capital expenses.

83. Which of the following recommendations related to disinfection and sterilization in healthcare facilities is a CDC category 1A recommendation? 1) "Before use on each patient, sterilize critical medical and surgical devices and instruments that enter normally sterile tissue or the vascular system or through which a sterile body fluid flows 2) "Meticulously clean patient-care items with water and detergent, or with water and enzymatic cleaners before high-level disinfection or sterilization procedures." 3) "In hospitals, perform most cleaning, disinfection, and sterilization of patient-care devices in a central processing department in order to more easily control quality." 4) "Perform low-level disinfection for noncritical patient-care surfaces (e.g., bedrails, over-the-bed table) and equipment (e.g., blood pressure cuff) that touch intact skin" a. 1 b. 1, 3 c. 1, 2, 4 d. 1, 2, 3, 4

83. A 1 Rationale: The CDC has established a system for cataloging recommendations based on the amount of data available to support the recommendation. Category 1A recommendations are strongly supported by epidemiologic, clinical, or experimental data from well-designed studies. Sterilization of medical instruments that will come into contact with sterile tissue or the vascular system is a Category 1A recommendation.

84. The IP is consulted prior to the construction of a new unit in her hospital. The design that is most concerning to her is: a. AII rooms (AIIR) that have negative air pressure b. Protective environment rooms that have positive air pressure c. Dual-purpose rooms that can alternate between negative and positive air pressure d. All rooms with neutral air pressure

84. C Dual-purpose rooms that can alternate between negative and positive air pressure Rationale: Isolation rooms can serve two purposes. The firt is to provide appropriate isolation for patients infected with pathogens that are transmitted by the airborne route (e.g., M. tuberculosis, varicella-zoster virus, rubeola [measles] virus). The major goal in this situation is to prevent transmission of pathogens from an infected patient to other patients, staff, or visitors. This is generally achieved by maintaining AIIRs. The second purpose is to provide a PE for severely immunosuppressed patients. AIIRs have negative air pressure, and PE rooms have positive air pressure with respect to adjacent areas. Correct direction of airflow and properly balanced air pressure in AIIR or PE areas are essential elements to consider during construction and renovation. The Facility Guidelines Institute's (FGI) Guidelines do not support "reversible" airflw rooms based on complexity of pressure relationships, concerns for serious patient and HCP outcomes if errors are made, and labor intensity needed for preventive maintenance.

85. Which of the following clinical uses would not be indicated for Gram stain results? a. Early identification of antibiotic resistance b. Initial direction for empiric antibiotic treatment c. Quality of the specimen d. Need for Isolation Precautions

85. A Early identification of antibiotic resistance Rationale: Gram staining of a specimen may help to determine the quality of a specimen, initial direction for therapy (empiric antibiotics), or the need for Isolation Precautions (e.g., Gram-negative diplococci in CSF, suggesting meningococci). It does not identify antibiotic resistance.

87. An IP recommends that chlorhexidine gluconate (CHG) be used to prepare a site for the insertion of a peripherally inserted central catheter (PICC) because it: 1) Requires only 30 seconds to dry 2) Can be used for all ages 3) Is not associated with allergic reactions 4) Can be easily rinsed after the line is inserted a. 3, 4 b. 1, 3 c. 2, 3 d. 1, 4

87. B 1, 3 Rationale: CHG products are increasingly used because of their effectiveness, 30-second dry time, and low incidence of allergic reactions. CHG products do not need to be removed or rinsed from the skin following venipuncture. CHG is not approved for use with infants younger than 2 months of age.

88. The laboratory runs a PCR test for MRSA. Both the positive and the negative controls amplify MRSA DNA. What can the IP conclude about this test? a. It is accurate because the positive control amplified MRSA DNA b. It is accurate because the negative control amplified MRSA DNA c. It is inaccurate because the positive control amplified MRSA DNA d. It is inaccurate because the negative control amplified MRSA DNA

88. D It is inaccurate because the negative control amplified MRSA DNA Rationale: PCR is an automated technique used to detect a target sequence of DNA that is unique to an organism. Positive and negative PCR controls are designed to monitor assay performance. The positive control is intended to monitor for substantial reagent failure. The negative control is used to detect reagent or environmental contamination by either MRSA DNA or MRSA amplicons. The negative control should not produce any amplifiction signal. Amplifiction of MRSA DNA in this control indicates that there was contamination of the samples with MRSA DNA, so the test results are not accurate.

89. Immunoglobulins are available for which of the following? 1) Tetanus 2) Rubella 3) Pertussis 4) Varicella-zoster a. 1, 2, 3 b. 2, 3, 4 c. 1, 3, 4 d. 1, 2, 4

89. D 1, 2, 4 Rationale: Immunoglobulins are special concentrated antibody preparations that provide immediate short-term protection against disease for individuals who are at high risk of experiencing severe disease or of developing serious complications from the disease. Human normal immunoglobulin preparations for Hepatitis A, measles, polio, and rubella, and specific immunoglobulinpreparations for Hepatitis B, rabies, and varicella-zoster for intramuscular use, are available.

90. The Director of Infection Prevention and Control has been asked to lead a team in the development of an antimicrobial stewardship program at an organization. Core members should include: 1) Pharmacist 2) Risk manager 3) Respiratory therapist 4) Infectious disease physician a. 1, 2 b. 1, 4 c. 3, 4 d. 2, 3

90. B 1, 4 Rationale: Multidisciplinary groups, including pharmacists, should establish a system for monitoring resistance and antibiotic usage, establish practice guidelines and other polices to control the use of antibiotics, respond to data from the monitoring system, and measure outcomes to evaluate the effectiveness of policies. Ideally, core members of an antimicrobial stewardship team include an infectious diseases physician, a clinical pharmacist with infectious diseases training, a clinical microbiologist, an information system specialist, an IP, and a hospital epidemiologist.

91. The critical care classes have 48 new students. There are eight males. The ratio of females to males is: a. 1:5 b. 5:1 c. 6:1 d. 1:6

91. B 5:1 Rationale: A ratio is a relationship between two numbers of the same kind expressed as "a to b" or a:b. It provides a comparison of two quantities. The ratio of females to males is 40:8 or 5:1.

92. A patient with a positive HBsAg must be dialyzed. Which of the following are the best measures to prevent transmission of disease? 1) Perform treatment in a separate room or area 2) Use a dedicated machine and equipment 3) Perform the treatment at the end of the day 4) Schedule the treatment on a separate day with other positive HBsAg patients a. 1, 2 b. 2, 4 c. 3, 4 d. 1, 3

92. A 1, 2 Rationale: The CDC recommends separating HBsAg patients by room or area and using a separate, dedicated machine and equipment to reduce the risk of transmission of HBV in the dialysis setting. Patients who are known to be positive for HBsAg should be excluded from reprocessing programs because of the risk of transmission to susceptible reuse personnel. The incidence of HBsAg has been found to be higher in dialysis units that do not follow recommendations.

93. Which of the following tasks would require wearing sterile gloves? a. Central line dressing change b. Routine dressing change without debridement c. Wound cleaning d. Intramuscular (IM) injection

93. A Central line dressing change

94. In a case-control study of the association between exposure to long-term nursing care and the outcome of VRE colonization, the authors report an odds ratio of 2.3. How should this odds ratio be interpreted? a. The study is flawed because a relative risk (RR) should have been calculated b. The risk of being placed in long-term nursing care is 2.3 times higher for people with VRE than without VRE c. VRE cases have 2.3 times the odds of having had exposure to long-term nursing care than non-VRE cases d. There is no association between VRE colonization and exposure to long-term nursing care

94. C VRE cases have 2.3 times the odds of having had exposure to long-term nursing care than non-VRE cases Rationale: A case-control study groups participants by their disease status (in this example, VRE-colonized cases or VRE-noncolonized controls) and looks retrospectively to determine whether they had an exposure of interest. The odds ratio that is calculated from a case-control study is the odds of the cases having had the exposure divided by the odds of the controls having had the exposure. Although this is often interpreted the same way as an RR, a case-control study does not give information about risk because incidence rates are not calculated in this study design. The odds ratio is not the risk of developing the disease given the exposure, but rather the ratio of the odds of having had the exposure given the disease status.

95. A used endoscope was placed in a carrying case before being cleaned and reprocessed. Which of the following actions should be taken? a. The endoscope should be cleaned and reprocessed and placed back in the carrying case b. The carrying case should be retained and only utilized for endoscopes that have not yet been cleaned or reprocessed c. The carrying case should be discarded d. The carrying case should be retained and only used for endoscopes that have been wrapped after cleaning and disinfection

95. C The carrying case should be discarded Rationale: Of all medical instruments, endoscopes have the highest association with healthcare outbreaks. Endoscope carrying cases should not be used to store clean or dirty endoscopes within a healthcare facility. If an unprocessed endoscope is placed in a carrying case then the case should be discarded because of the potential for a clean endoscope to become contaminated through contact with the case.

96. Based on current CDC guidelines, which of the following modes of transmission is most responsible for contamination of a central venous catheter? a. Airborne b. Direct contact c. Indirect contact d. Droplet spread

96. B Direct contact Rationale: According to the CDC Guidelines for the Prevention of Intravascular Catheter-Related Infections, there are four recognized routes for contamination of catheters: (1) migration of skin organisms at the insertion site into the cutaneous catheter tract and along the surface of the catheter with colonization of the catheter tip; this is the most common route of infection for short-term catheters; (2) direct contamination of the catheter or catheter hub by contact with hands or contaminated fluids or dvices; (3) less commonly, catheters might become hematogenously seeded from another focus of infection; and (4) rarely, infusate contamination might lead to catheter-related bloodstream infections.

97. The IP has identified poor ompliance with hand hygiene practices in one of the critical care units. She has discussed her observations with the management of the unit and together they have identified a solution o improve practices. To ensure staff's adherence to the solution, in which of the activities below should the IP engage before implementing the solution? a. Perform a survey of fellow IPs to discuss the strategies they have found to be effective and that have sustained acceptable compliance b. Form a focus group of unit staff to discuss her observations, the ideas to improve compliance, and the proposed solution c. Develop a strategy with management to penalize any unit staff observed to be noncompliant d. Each month post the names of staff members she observes to be noncompliant with hand hygiene practices

97. B Form a focus group of unit staff to discuss her observations, the ideas to improve compliance, and the proposed solution Rationale: Focus groups provide an opportunity for investigators to explore the beliefs of participants and provide an avenue for perceptions and concerns to be identified and addressed. Focus groups are a qualitative research method that can be used in quality improvement initiatives. For example, focus groups are used to obtain reactions to proposed changes or proposed solutions to problems, to describe perspectives that may differ from the researcher's, to describe relationships within groups (e.g., coping strategies), to assess programs and outcomes of services, and to confirm hypotheses.

98. Which of the following patients would have the lowest risk of SSI? a. A 53-year-old male with insulin-dependent diabetes and coronary artery disease undergoing elective aortofemoral bypass b. A 38-year-old female with mild but controlled hypertension undergoing a laparoscopic cholecystectomy c. A 42-year-old, well-conditioned male undergoing elective groin hernia repair d. A 62-year-old female on chronic renal hemodialysis undergoing emergency laparotomy for perforated diverticulitis

98. C A 42-year-old, well-conditioned male undergoing elective groin hernia repair Rationale: Elective procedures carry a lower risk for postoperative infection than urgent ones. It is important to consider these factors when making decisions regarding surgical outcomes and quality improvement assessments. A surgical risk index is a score used to predict a surgical patient's risk of acquiring a surgical site infection. The risk index score, ranging from 0 to 3, is the sum of the number of risk factors present among the following: • Surgical site wound classification of contaminated or dirty (class III or IV)• American Society of Anesthesiology (ASA) score as rated by an anesthesiologist before operation of ≥ 3 • Prolonged procedure time, where the threshold in minutes (i.e., the cut point) is above the 75th percentile of the duration of surgery for the specific procedure being performed as determined by the NHSN database The higher the score by this index, the greater is the risk for subsequent SSI.

99. What organism is most often associated with urinary tract infections? a. Enterococcus sp. b. Candida sp. c. Klebsiella pneumoniae or K. oxytoca d. Escherichia coli

99. D Escherichia coli Rationale: In the United States, urinary tract infections account for about 4 million ambulatory care visits each year, representing about 1 percent of all outpatient visits. Gram-negative bacilli and enterococci are the primary enteric bacteria that can grow in human urine. The presence of bacteria in urine almost always precedes intestinal colonization by the infecting bacteria. E. coli is the predominant aerobic Gram-negative organism of normal bowel flora, and thus is the most common organism isolated from urinary tract infections

8. Frequency histograms and polygons are most useful for variables of what level of measurement? 1) Interval scale 2) Ratio scale 3) Nominal scale 4) Ordinal scale a. 1, 2 b. 1, 3 c. 2, 3 d. 3, 4

A 1, 2 Rationale: A histogram is a graphic of a frequency distribution in which one bar is used for each time interval, and there is no space between the intervals (see Figure PE3-1). It is used to portray the (grouped) frequency distribution of a variable at the interval or ratio level of measurement.

64. A patient is suspected of having bacterial meningitis. A CSF sample was taken and had the following results: opening pressure was normal, glucose concentration was normal, lymphocytes were the predominant inflammatory cell, white blood cell (WBC) count was 88 per cubic mm, protein was normal, and the Gram and AFB stains were negative. Given these results, which of the following organisms could be the cause of the meningitis? a. Herpes simplex virus b. Haemophilus influenza c. Mycobacterium avium d. Streptococcus pneumonia

A Herpes simplex virus

9. Which of the following is the correct order for disinfecting endoscopes? a. High-level disinfection, rinse the scope with sterile water, flush the channels with 0 to 90 percent alcohol, dry using forced air b. Rinse the scope using sterile water, high-level disinfection, flush the channels with 0 to 90 percent alcohol, dry using forced air c. Flush the channels with 70 to 90 percent alcohol, rinse the scope using sterile water, high-level disinfection, dry using forced air d. Dry using forced air, high-level disinfection, flush the channels with 70 to 90 percent alcohol, rinse the scope using sterile water

A High-level disinfection, rinse the scope with sterile water, flushthe channels with 70 to 90 percent alcohol, dry using forced air Rationale: Cleaning is essential before manual or automated disinfection. Thoroughly clean the entire endoscope immediately after use, then completely immerse the endoscope and endoscope components in the high-level disinfectant/sterilant and ensure that all channels are perfused. After high-level disinfection, rinse the endoscope and flush the channels with sterile, filered, or tap water to remove the disinfectant/sterilant. Discard the rinse water after each use/cycle. Flush the channels with 70 to 90 percent ethyl or isopropyl alcohol and dry using forced-air. The final drying teps greatly reduce the possibility

22. Which of the following is conducted to reduce construction hazard risk? a. Mitigation b. Preparedness c. Response d. Recovery

A Mitigation Rationale: An ICRA is a multidisciplinary, organizational, documented process that after considering the facility's patient population and program:• Focuses on reduction of risk from infection,• Acts through phases of facility planning, design, construction, renovation, facility maintenance, and• Coordinates and weighs knowledge about infection, infectious agents, and care environment, permitting the organization to anticipate potential impact.The ICRA elements are divided into three areas: processes for design, construction, and mitigation. Mitigation recommendations should address the following:• Patient placement and relocation.• Standards for barriers and other protective measures required to protect adjacent areas and susceptible patients from airborne contaminants.• Temporary provisions or phasing for construction or modification of heating, ventilation, air conditioning, and water supply systems.• Protection from demolition.• Measures taken to train hospital staff, visitors and construction personnel.

3. After an incident in which human milk from one mother was mistakenly fed to another mother's infant, the infection preventionist (IP) is tasked with leading a team to determine why the event occurred and how it can be prevented from happening again in the future. This type of analysis is known as: a. Root cause analysis (RCA) b. Gap analysis c. Strengths-weaknesses-opportunities-threats (SWOT) analysis d. Failure mode effect analysis (FMEA)

A Root cause analysis Rationale: RCA, gap analysis, SWOT analysis, and FMEA are tools that can be used to improve quality. The RCA process takes a retrospective look at adverse outcomes and determines what happened, why it happened, and what an organization can do to prevent the situation from recurring. Gap analysis is a tool that is used to take an organization from a current state to a future state where organizational objectives are met. A SWOT analysis is a process where the organization or group assesses their own positive and negative points, outlines opportunities for improvement and growth, and lists any threats that might impede those opportunities. A FMEA ca

6. A patient is admitted with fever, nausea and vomiting, sensitivity to light, and stiff neck. Symptom onset has been progressing slowly over several weeks. The Gram stain of the cerebrospinal fluid CSF) is India ink positive, and there is low glucose and predominant lymphocytes. What type of precautions is required for this patient? a. Standard plus Droplet Precautions b. Standard Precautions c. Standard plus Airborne Precautions d. Standard plus Contact Precautions

B Standard Precautions Rationale: Cryptococcosis is the most common fungal infection of the central nervous system. Cryptococcal meningitis is caused by the fungus Cryptococcus neoformans. This fungus is found in soil around the world. Cryptococcosis is believed to be acquired by inhalation of the fungus from the environment. Unlike bacterial meningitis, this form of meningitis comes on more slowly, over a few days to a few weeks. India ink will stain the polysaccharide capsule of C. neoformans, and C. neoformans meningitis is characterized by low glucose in the CSF and predominant lymphocytes. These results allow fungal meningitis to be distinguished from (1) bacterial meningitis, which is characterized by fairly normal glucose levels and predominant neutrophils; (2) viral meningitis, which is stain negative with normal glucose; and (3) mycobacterial meningitis, which is AFB-stain positive. Meningitis with C. neoformans is not communicable so only Standard Precautions are needed in this case.

11. An IP wants to make external comparisons of the data collected relating to catheter-associated urinary tract infections (CAUTI) in the Intensive Care Unit (ICU). Which of the formulas below will allow the IP to accurately benchmark against nationally available data? a. The number of ICU patients with urinary catheters in a given month divided by the number of urinary tract infections identified in the same month times 1000 b. The number of ICU urinary tract infections identified in a given month divided by the number of urinary catheter days in the same month times 1,000 c. The number of urinary catheters used in the ICU in a given month divided by the number of urinary tract infections identified in the same month times 1000 d. The number of urinary tract infections identified in a given month divided by the total number of patient days in the ICU in the same month times 1,000

B The number of ICU urinary tract infections identified in a given month divided by the number of urinary catheter days in the same month times 1,000 Rationale: Data should be expressed as rates or ratios that are calculated using the same methodology as a nationally validated surveillance system. This allows an organization to compare its rates with another organization or a recognized benchmark. The NHSN indicator for CAUTI measures the development of a urinary tract infection associated with the risk of an indwelling urinary catheter in a defined popultion. The CAUTI rate is calculated as (# CAUTIs identified÷ # indwelling catheter days) × 1,000.

10. The lead IP is invited to participate in an employee health process improvement project. The goal of the project is to improve influenza accination rates among employees. A multidisciplinary team is formed with representation from front-line employees. After conducting a root cause analysis (RCA), the team develops a proposal for improvement to present to the facility leadership team. Which of the following elements should be included as part of the proposal to most clearly communicate to the leadership team the factors contributing to the facilities' current vaccination rate among employees? a. A timeline for implementing improvement activities b. A Pareto chart c. A fishbone diagram with an explanation d. A strategic plan

C A fishbone diagam with an explanation Rationale: The RCA process takes a retrospective look at adverse outcomes and determines what happened, why it happened, and what an organization can do to prevent the situation from recurring in the future. The product of the RCA is an action plan that identifies the trategies that the organization intends to implement to improve safety. A thorough RCA demonstrates credibility of the recommended process to the facility leadership team. When used during the RCA process, a fishbone diagam (also called an Ishikawa diagram) will help identify and visually display both the elements involved in the improvement project and the areas of responsibility and accountability (see Figure PE3-3). Used in conjunction with the RCA, the fishbone diagram with an accompanying explanation present a clear picture of the both improvement project and rationale behind it.

2. The most unlikely method of transmission of infection in a healthcare setting would be: a. Airborne b. Vehicle c. Vector d. Contact

C Vector Rationale: Vectors, such as insects, may transmit infectious organisms in the healthcare setting; however, this method of transmission is of less importance in most industrialized nations. External vector-borne transmission is the mechanical transfer of microorganisms by a vector, such as a fly on ood. Internal vector-borne transmission involves transfer of infectious material directly from the vector into the new host, such as occurs in mosquitoes and malaria, fleas and plagu, and louse-borne typhus. The vector may simply harbor the infectious organism, with no biological interaction taking place, or the agent may actually undergo changes within the vector (e.g., malaria parasites require that part of their life cycle take place within a mosquito).

5. Which of the following aid in the diagnosis of pseudomembranous colitis (PMC)? 1) Colonoscopic biopsy of lesions 2) Raised antibody levels in blood to Clostridium difficil toxin 3) Positive blood culture for Clostridium difficil 4) Stool assays for Clostridium difficil a. 1, 2 b. 2, 3 c. 3, 4 d. 1, 4

D 1, 4 Rationale: Pseudomembranous colitis (PMC) is an acute colitis characterized by the formation of an adherent inflammatory membrane (pseudomembrane) overlying sites of mucosal injury. Clostridium difficil infection is responsible for the majority of cases of PMC and for as many as 20 percent of cases of antibiotic-induced diarrhea without colitis. The method of choice fore establishing the diagnosis is done by stool assays for C. difficil toxins or by colonoscopy.

4. Which one of the following best describes the difference between a common point source outbreak and a propagated outbreak? a. Case fatality rates in common source outbreaks are higher b. Person-to-person transmission is a feature of common source outbreaks c. The attack rate in propagated outbreaks is higher d. All cases in a common point source outbreak occur within one incubation period of the exposure

D All cases in a common point source outbreak occur within one incubation period of the exposure Rationale: A common source epidemic is characterized by a rapid spread with cases presenting at the same stage of the disease, indicating the single source of the pathogen. Propagated outbreaks are outbreaks in which the disease propagates in one or more initial cases and then spreads to others, a relatively slow method of spread.

7. How can streptococci be differentiated from staphylococci under the microscope following the Gram stain procedure? a. Staphylococci are Gram positive cocci, while streptococci are Gram positive bacilli b. Staphylococci are Gram negative bacilli, while streptococci are Gram negative cocci c. Staphylococci are Gram positive, while streptococci are Gram negative d. Staphylococci are Gram positive and grow in grape-like clusters, while streptococci which are also Gram positive, grow in chains

D Staphylococci are Gram positive and grow in grape-like clusters, while streptococci, which are also Gram positive, grow in chains Rationale: Both staphylococci and streptococci have round, spherical cell shapes, but the arrangement of cells is different due to a different binary fision. Streptococci form a chain of round cells, because their division occurs in one linear direction, while staphylococci divide in various directions forming grape-like clusters. Both are facultatively anaerobic gram-positive bacteria.

revisit

revisit


Related study sets

3.4 Explain the importance of secure staging deployment concepts

View Set

Chapter 6: Life Premiums and Benefits

View Set

8th Grade - Chapter 26 - The Great Depression and New Deal

View Set

Biology 1202: MasteringBiology Chapter 23

View Set

BA 360 Intro to Financial Management- Ch 14

View Set

T/F HS 321 Income Taxation Chapter 16

View Set

prepU ch 47 Management of Patients With Intestinal and Rectal Disorders

View Set